Đến nội dung

angleofdarkness nội dung

Có 245 mục bởi angleofdarkness (Tìm giới hạn từ 29-04-2020)



Sắp theo                Sắp xếp  

#495207 Topic về Bất đẳng thức, cực trị THCS

Đã gửi bởi angleofdarkness on 26-04-2014 - 11:59 trong Bất đẳng thức và cực trị

Câu 2

Trong cuốn “Sáng tạo bất đẳng thức” có một bài toán thế này

“Chứng minh với mọi số thực dương a, b, c ta có

$\frac{{{a}^{3}}b}{1+a{{b}^{2}}}+\frac{{{b}^{3}}c}{1+b{{c}^{2}}}+\frac{{{c}^{3}}a}{1+c{{a}^{2}}}\ge \frac{abc\left( a+b+c \right)}{1+abc}$.

Lời giải như sau:

Với mọi số thực dương k, áp dụng BĐT Cauchy - Schwarz ta có

$\frac{{{a}^{2}}}{b+kc}+\frac{{{b}^{2}}}{c+ka}+\frac{{{c}^{2}}}{a+kb}\ge \frac{a+b+c}{k+1}$

Ta chọn $k=\frac{1}{abc}$ thì có được đpcm.”

Mình xin hỏi tại sao lại nghĩ đến BĐT $\frac{{{a}^{2}}}{b+kc}+\frac{{{b}^{2}}}{c+ka}+\frac{{{c}^{2}}}{a+kb}\ge \frac{a+b+c}{k+1}$, và chọn $k=\frac{1}{abc}$.

 

 

 

Ta thấy tử là $a^3b$ và mẫu chứa $a^2b$; hai số này có nhân tử chung là ab nên chia cả tử và mẫu cho ab sẽ gọn hơn.

 

Như vậy ta sẽ có $\dfrac{{{a}^{3}}b}{1+a{{b}^{2}}}=\dfrac{a^2}{\dfrac{1}{ab}+b}$ 

 

Tương tự thì $\dfrac{{{b}^{3}}c}{1+b{{c}^{2}}}=\dfrac{b^2}{\dfrac{1}{bc}+c}$ và $\dfrac{{{c}^{3}}a}{1+c{{a}^{2}}}=\dfrac{c^2}{\dfrac{1}{ca}+a}$

 

Đến lúc này thì có thể dùng BĐT Cauchy - Schwarz rồi, mẫu là $\dfrac{1}{ab}+b +\dfrac{1}{bc}+c +\dfrac{1}{ca}+a$

 

Nhưng để cho gọn (nhóm mẫu mới vào sao cho có hạng tử a + b + c ) thì cần biến đổi $\dfrac{1}{ab}=\dfrac{c}{abc}$, các hạng tử này đã đồng nhất 

 

Xong.




#495209 Topic về Bất đẳng thức, cực trị THCS

Đã gửi bởi angleofdarkness on 26-04-2014 - 12:05 trong Bất đẳng thức và cực trị


Câu 3:

Trên mathlinks.ro có bài toán sau:

“Cho các số thực dương a, b, c thỏa $3a+4b+6c\ge 42$. Chứng minh

$a+b+c+\frac{3}{a}+\frac{6}{b}+\frac{8}{c}\ge \frac{29}{2}$.

Lời giải trên trang ấy là

BĐT đề bài tương đương $\frac{3{{\left( a-2 \right)}^{2}}}{4a}+\frac{2{{\left( b-3 \right)}^{2}}}{3b}+\frac{{{\left( c-4 \right)}^{2}}}{2c}+\frac{3a+4b+6c-42}{12}\ge 0$ (1) ”

hãy chỉ mình phương pháp tìm ra (1).

Mình sẽ rất vui nếu như bạn trả lời các câu hỏi của mình. 

 

Chỉ là chuyển $\dfrac{29}{2}$ sang VT để có VT $\geq 0$ cho gọn và biến đổi tương đương thành tổng các phân số sao cho có phân số chứa tử là 3a + 4b + 6c - 42 (cái cụm này đã $\geq 0$)

 

Các số còn thừa lại nhóm thích hợp theo cụm chứa a, b, c để tạo HĐT là đc.

 




#495205 Topic về Bất đẳng thức, cực trị THCS

Đã gửi bởi angleofdarkness on 26-04-2014 - 11:45 trong Bất đẳng thức và cực trị

Mình có một số vấn đề về BĐT xin bạn giúp đỡ.

Câu 1: Trong cuốn sách Phương pháp giải toán BĐT và cực trị dành cho học sinh 8, 9

Cho 3 số thực không âm a, b, c. Chứng minh rằng

${{\left( {{a}^{2}}+{{b}^{2}}+{{c}^{2}} \right)}^{2}}\ge 4\left( a+b+c \right)\left( a-b \right)\left( b-c \right)\left( c-a \right)$.

Lời giải của bài này trong cuốn sách như sau

Do vai trò của a, b, c có tính hoán vị vòng qubạn nên giả sử b nằm giữa a và c.

Nếu $a\ge b\ge c$ thì vế phải của BĐT âm, còn vế trái dương nên BĐT hiển nhiên.

Nếu $c\ge b\ge a$, ta có

$VP=4\left( a+b+c \right)\left( b-a \right)\left( c-b \right)\left( c-a \right)\le {{\left[ \left( a+b+c \right)\left( b-a \right)+\left( c-a \right)\left( c-b \right) \right]}^{2}}$

Ta chỉ cần chứng minh ${{a}^{2}}+{{b}^{2}}+{{c}^{2}}\ge \left( a+b+c \right)\left( b-a \right)+\left( c-b \right)\left( c-a \right)$. BĐT này tương đương với $a\left( 2a+2c-b \right)\ge 0$, đúng do $c\ge b\ge a$.

Mình xin hỏi:”Tại sao lại tách và sử dụng BĐT như vầy

$VP=4\left( a+b+c \right)\left( b-a \right)\left( c-b \right)\left( c-a \right)\le {{\left[ \left( a+b+c \right)\left( b-a \right)+\left( c-a \right)\left( c-b \right) \right]}^{2}}$

Tác giả đã dự đoán dấu bằng ra sao mà lại nhóm như thế? Đã sử dụng kỹ năng nào?

Mình đã thử làm khác đi là: giả sử $a=\max \left\{ a,b,c \right\}$. Nếu $b\ge c$ thì BĐT hiển nhiên. Xét $c\ge b$, sau đó mình làm như sau

$VP=4\left( a+b+c \right)\left( a-c \right)\left( c-b \right)\left( a-b \right)\le {{\left[ \left( a+b+c \right)\left( a-c \right)+\left( a-b \right)\left( c-b \right) \right]}^{2}}$

Ta chỉ cần chứng minh ${{a}^{2}}+{{b}^{2}}+{{c}^{2}}\ge \left( a+b+c \right)\left( a-c \right)+\left( a-b \right)\left( c-b \right)$. BĐT này tương đương với $c\left( 2c+2b-a \right)\ge 0$. Điều này là không chắc chắn.

Bạn hãy giải thích sai lầm của mình ở chỗ nào nhé.

Mình xin cảm ơn

 

 

 

- Thứ nhất: BĐT tác giả đã dùng là hệ quả của BĐT Cauchy 2 số: với x, y không âm (tức $\geq 0$) thì $4xy \leq x^2+y^2$ (*)

 

- Thứ hai: Mấy bài BĐT mà các biến tham gia có vai trò như nhau (hoán vị vòng quanh) thì dấu = xảy ra thường là các biến bằng nhau. Áp dụng đó để nhóm số cho phù hợp. Bạn có thể tham khảo kĩ thuật sử dụng BĐT Cauchy. (sách hoặc search)

 

- Thứ ba: cách khác của bạn chỉ là suy ra từ cách của tác giả, dĩ nhiên vẫn phải lựa chọn để nhóm các hạng tử mà sử dụng BĐT (*) kia vẫn đúng. Ở đây mình sửa theo cách của bạn là:

 

$VP=4\left( a+b+c \right)\left( a-c \right)\left( c-b \right)\left( a-b \right)\le {{\left[ \left( a+b+c \right)\left( c-b \right)+\left( a-b \right)\left(  a-c \right) \right]}^{2}}$

 

Ta chỉ cần chứng minh ${{a}^{2}}+{{b}^{2}}+{{c}^{2}}\ge {{\left[ \left( a+b+c \right)\left( c-b \right)+\left( a-b \right)\left(  a-c \right) \right]}^{2}}$. BĐT này tương đương với $b\left( 2c+2a-c \right)\ge 0$.

 

BĐT này đúng do $a \geq c \geq b$

 




#495285 Topic về Bất đẳng thức, cực trị THCS

Đã gửi bởi angleofdarkness on 26-04-2014 - 18:59 trong Bất đẳng thức và cực trị

Cho x,y>0 và x+y=1.Cm 

3(3x-2)$^{2}$+ $\frac{8x}{y}$$\geq$7

 

 

Cho x,y,z$\in [1;2]$. Tìm max

A=(x+y+z)($\frac{1}{x}+\frac{1}{y}+\frac{1}{z}$)

 

Bài 1/

 

Bạn tham khảo các cách làm ở pic này




#495289 Topic về Bất đẳng thức, cực trị THCS

Đã gửi bởi angleofdarkness on 26-04-2014 - 19:25 trong Bất đẳng thức và cực trị

Cho x,y,z$\in [1;2]$. Tìm max

A=(x+y+z)($\frac{1}{x}+\frac{1}{y}+\frac{1}{z}$)

 

untitled38.png

 

Mình chụp hình lời giải (bàn phím lỗi k gửi đc bài -_- )

 

P/S: mình đổi nhầm biến x; y; z thành a; b; c :P Bạn thông cảm!




#485995 Cơ bản về nguyên lý Đi-rích-lê

Đã gửi bởi angleofdarkness on 06-03-2014 - 00:04 trong Toán rời rạc


1.Cho 2 số tự nhiên a,b sao cho $ab=1996^{1995}$. Hỏi $a+b$ có chia hết cho $1995$ hay không?

 

 

Bài này là bài 6 trong đề thi cuối HKI huyện Thanh Chương - Nghệ An, bạn tham khảo đáp án ở đây

 

2. Trong hình chữ nhật có kích thước $1\times 2$, lấy $6n^{2}+1$ điểm với n nguyên dương. Chứng minh rằng tồn tại 1 hình tròn bán kính $\frac{1}{n}$ chứa không ít hơn 4 trong số các điểm đã cho

 

Còn bài này là đề thi HSG 9 trường Nguyễn Trãi đáp án tham khảo ở đây 

 

:D




#485994 Cơ bản về nguyên lý Đi-rích-lê

Đã gửi bởi angleofdarkness on 06-03-2014 - 00:03 trong Toán rời rạc

 

Bài này nha

Một cuộc họp có 100 người tham gia . Giả sử mỗi người đều quen với ít nhất 67 người. CMR: có thể tìm thấy 1 nhóm 4 người mà 2 người bất kì đều quen nhau.

 

Xét hai người quen nhau A và B. Số người quen của mỗi người ít nhất là 67 nên số người quen chung của A và B ít nhất là 67 + 67 - 100 = 34.

 

Gọi S là tập những người quen chung của A và B thì trong M có ít nhất hai người C và D quen nhau vì nếu C và D không quen nhau thì mỗi nguời trong nhóm S phải quen nhiều nhất là 100 - 34 = 66 người (ít hơn 67) nên loại.

 

Vậy có ít nhất 4 người (VD là A, B, C, D như trên) mà bất kì người nào trong nhóm cũng quen nhau.

 




#491045 [TOPIC] Bài toán tính tổng các dãy số có quy luật

Đã gửi bởi angleofdarkness on 06-04-2014 - 14:50 trong Đại số

cho em hỏi 1 bài 

Căn bậc 2 của 24 - căn bậc 2 của 23 + căn bậc 2 của 22 -......- căn bậc 2 của 3 +  căn bậc 2 của 2 -  căn bậc 2 của 1 

chứng minh nó <5/2

 

Ta có $\sqrt{24}-\sqrt{23}+\sqrt{22}-\sqrt{21}+...+\sqrt{2}-\sqrt{1} \\ =\dfrac{1}{\sqrt{24}+\sqrt{23}}+\dfrac{1}{\sqrt{22}+\sqrt{21}}+...+\dfrac{1}{\sqrt{2}+\sqrt{1}} \\ \leq \dfrac{1}{4}. \Big( \dfrac{1}{\sqrt{24}}+\dfrac{1}{\sqrt{23}}+\dfrac{1}{\sqrt{22}}+\dfrac{1}{\sqrt{21}}+...+\dfrac{1}{\sqrt{2}}+\dfrac{1}{\sqrt{1}} \Big) \\ < \dfrac{1}{4}.(2.\sqrt{24}) \\ < \dfrac{1}{4}.(2.\sqrt{25})=\dfrac{5}{2}$




#487338 $\boxed{\text{Chuyên Đề}}$ Bất đẳng thức - Cực trị

Đã gửi bởi angleofdarkness on 17-03-2014 - 11:58 trong Bất đẳng thức và cực trị

Bài 1: Cho $a,b,c$ là các số thực không âm. CMR:

$\sum \frac{(b+c-a)^2}{2a^2+(b+c)^2}\geq \frac{3.\sum a^2}{(\sum a)^{2}}$

Bài 2: Cho $a,b,c$ là các số thực dương thỏa. Tìm max của:

$\sum \frac{a(b+c)}{(b+c)^2+a^2}$

 

Bạn xem lại đề giúp mình cả dấu $\geq$ hay $\leq$ và xem thỏa mãn gì với :D




#488678 $\boxed{\text{Chuyên Đề}}$ Bất đẳng thức - Cực trị

Đã gửi bởi angleofdarkness on 25-03-2014 - 11:44 trong Bất đẳng thức và cực trị

$min của \sum \frac{x^{4}}{(x^{2}+y^{2})(x+y)} với x+y+z=3$ 

 

 

Bài 120:

Cho $x,y>0$ và $x+y=2$,. Tìm max $A=2xy(x^{2}+y^{2})$

 

Đề nghị đăng đúng STT bài để tiện theo dõi,




#484777 $\boxed{\text{Chuyên Đề}}$ Bất đẳng thức - Cực trị

Đã gửi bởi angleofdarkness on 25-02-2014 - 18:40 trong Bất đẳng thức và cực trị

96/ Cho $0<x;y;z \leq \frac{3}{2}$. Chứng minh rằng $\sum \sqrt{x^2+\frac{1}{x^2}} \geq \frac{3}{2}.\sqrt{17}$

 

97/ Cho x; y; z > 0. Chứng minh $\sum \frac{2x}{x^6+y^4} \leq \sum \frac{1}{x^4}$

 

98/ Cho x; y; z > 0 và $\sum \frac{a^5}{b+c}=\frac{3}{2}$. Chứng minh $\sum ab^2 \leq 3$

 

99/ Cho $a;b;c>0$ và $a+b+c \leq \frac{3}{2}$. Chứng minh $\sum (a+\frac{1}{b})^3 \geq \frac{375}{8}$

 

100/ Cho x; y; z > 0. Chứng minh $\sum \frac{x^7}{x^2+y^2} \geq \frac{\sum x^5}{2}$




#487740 $\boxed{\text{Chuyên Đề}}$ Bất đẳng thức - Cực trị

Đã gửi bởi angleofdarkness on 19-03-2014 - 11:11 trong Bất đẳng thức và cực trị

Bài 3: Cho $a,b,c$ là các số thực dương. CMR

$\sum \frac{(b+c-a)^2}{2a^2+(b+c)^2}\geq \frac{1}{2}$

 

 

$\sum \dfrac{(b+c-a)^2}{2a^2+(b+c)^2}\geq \dfrac{[\sum (b+c-a)]^2}{\sum [2a^2+(b+c)^2]}=\dfrac{(\sum a)^2}{2\sum a^2+\sum (b+c)^2} \\ \geq \dfrac{(\sum a)^2}{2\sum a^2+\sum [2(b^2+c^2)]}=\dfrac{(\sum a)^2}{6\sum a^2} \\ \geq \dfrac{3 \sum ab}{6\sum ab}=\dfrac{1}{2}$

 

:P

 

P/S: lần sau bạn chú ý đặt đề theo STT đã có trong pic bạn nhé!




#484768 $\boxed{\text{Chuyên Đề}}$ Bất đẳng thức - Cực trị

Đã gửi bởi angleofdarkness on 25-02-2014 - 17:56 trong Bất đẳng thức và cực trị


92) Cho $x;y;z$ thỏa: $x^2+2y^2+2x^2z^2+y^2z^2+3x^2y^2z^2=9$. Tìm Min; Max $A=xyz$

 

 

92/ *N/x: $p^2$ chính đối với những dạng bài này thường là tìm cách lập một pt bậc II với ẩn mới lấy xyz làm tham số. Kĩ thuật này thuộc về phần phân tích đa thức thành nhân tử:

 

Ta có $x^2+2y^2+2x^2z^2+y^2z^2+3x^2y^2z^2=9 \Rightarrow 9 \geq 2|xyz|+2.2|xyz|+3x^2y^2z^2$ (Cauchy cho 2 số không âm $x^2;y^2z^2$ và $y^2;x^2z^2$)

 

Hay $9 \geq 6|A|+3A^2 \Rightarrow 3(A^2-|A|+3|A|-3) \leq 0 \Leftrightarrow (|A|-1)(|A|+3) \leq 0$

 

$\Rightarrow -3 \leq |A| \leq 1 \Rightarrow |A| \leq 1 \Rightarrow -1 \leq A \leq 1$

 

Vậy Min A = -1; Max A = 1.

 

Dấu = ở Min và Max khi $x^2+2y^2+2x^2z^2+y^2z^2+3x^2y^2z^2=9$ và $x^2=y^2z^2;y^2=x^2z^2$

 

A Min khi (x; y; z) = (-1; -1; -1); (-1 ; 1; 1); (1; -1; 1); (1; 1; -1).

 

A Max khi (x; y; z) = (-1; -1; 1); (-1; 1; -1); (1; -1; -1); (1; 1; 1).

 

P\S: Bài này tìm lắm nghiệm kinh mà toàn là lặp -_- mất thì giờ quá.

 




#482218 $\boxed{\text{Chuyên Đề}}$ Bất đẳng thức - Cực trị

Đã gửi bởi angleofdarkness on 09-02-2014 - 15:04 trong Bất đẳng thức và cực trị


12) Cho $a;b;c>0$ thỏa: $a+b+c=3abc$. Cmr: $\sum \frac{1}{a^3}\geq 3$

 

 

Ta có $a+b+c=3abc \Rightarrow \sum \frac{1}{ab}=3$

 

Có $\frac{1}{a^3}+\frac{1}{b^3}+1 \geq \frac{3}{ab} \Rightarrow 2.\sum \frac{1}{a^3}+3 \geq 3.\sum \frac{1}{ab}$

 

$\Rightarrow \sum \frac{1}{a^3} \geq 3$

 

Dấu = khi ... khi a = b = c = 1.

 

P/s: Ai post bài lần sau xét dấu = giúp vậy, vì nhiều T.h k xảy ra dấu =. 




#482222 $\boxed{\text{Chuyên Đề}}$ Bất đẳng thức - Cực trị

Đã gửi bởi angleofdarkness on 09-02-2014 - 15:12 trong Bất đẳng thức và cực trị


9) Cho $a;b;c;d>0$. Cmr: $\sum \sqrt{\frac{a}{b+c+d}} \geq 2$

14) Cho $a;b;c>0$ thỏa: $\sqrt{\frac{a}{b+2c}}+\sqrt{\frac{b}{a+2c}}+2\sqrt{\frac{c}{a+b+c}}\geq 2$

 

Áp dụng bài số 9 trên ta có: $\sqrt{\frac{a}{b+c+d}}+\sqrt{\frac{b}{c+d+a}}+\sqrt{\frac{c}{d+a+b}}+\sqrt{\frac{d}{a+b+c}} \geq 2$

 

Cho d = c, thay vào các mẫu thì ta được $\sqrt{\frac{a}{b+2c}}+\sqrt{\frac{b}{a+2c}}+2\sqrt{\frac{a}{a+b+c}} \geq 2$

 

Tương tự 9 thì ở đây dấu = cũng k xảy ra.




#482214 $\boxed{\text{Chuyên Đề}}$ Bất đẳng thức - Cực trị

Đã gửi bởi angleofdarkness on 09-02-2014 - 14:51 trong Bất đẳng thức và cực trị


9) Cho $a;b;c;d>0$. Cmr: $\sum \sqrt{\frac{a}{b+c+d}} \geq 2$

 

Ta có $a+b+c+d \geq 2\sqrt{a(b+c+d)} \Rightarrow \sqrt{\frac{a}{b+c+d}} \geq \frac{2a}{a+b+c+d}$ (Cauchy cho 2 số dương)

 

$\Rightarrow \sum \sqrt{\frac{a}{b+c+d}} \geq 2$ (cộng các phân số cùng mẫu lại)

 

Dấu = khi $\sum a = \sum(b+c+d)$, tức $a+b+c+d=3(a+b+c+d)$ (điều này k xảy ra khi a, b, c, d > 0)

 

Vậy $\sum \sqrt{\frac{a}{b+c+d}}>2$




#481780 $\boxed{\text{Chuyên Đề}}$ Bất đẳng thức - Cực trị

Đã gửi bởi angleofdarkness on 07-02-2014 - 22:15 trong Bất đẳng thức và cực trị

10) Cmr: $\sum \frac{1}{a^{3}+b^{3}+abc}\leq \frac{1}{abc}\forall a;b;c>0$

 

$a^3+b^3 \geq a^2b+ab^2=ab(a+b)$

 

$\Rightarrow a^3+b^3+abc \geq ab(a+b+c) \Rightarrow \frac{1}{a^3+b^3+abc} \leq \frac{1}{ab(a+b+c)}$

 

$\Rightarrow \sum \frac{1}{a^3+b^3+abc} \leq \sum \frac{1}{ab(a+b+c)} = \frac{1}{abc}$ (đưa $\frac{1}{a+b+c}$ ra ngoài rồi quy đồng mẫu các phân thức bên trong lên)




#481777 $\boxed{\text{Chuyên Đề}}$ Bất đẳng thức - Cực trị

Đã gửi bởi angleofdarkness on 07-02-2014 - 22:07 trong Bất đẳng thức và cực trị

Làm rõ hơn chút, chỗ đó là cauchy 3 số.

 

 

 

Đã chỉnh.

 


 

Bước cuối tắt.

 

 

Cauchy cho 3 số $\frac{a}{b};\frac{b}{c};\frac{c}{a}$ thì mình nghĩ không cần viết dài thêm???

 

 

 

 

 




#483801 $\boxed{\text{Chuyên Đề}}$ Bất đẳng thức - Cực trị

Đã gửi bởi angleofdarkness on 18-02-2014 - 10:17 trong Bất đẳng thức và cực trị


71) Cho $a;b;c$ là 3 cạnh một tam giác. Cmr: $1\leq \sum \frac{a}{b+c}<2$

 

 

Do a, b, c là độ dài ba cạnh tam giác nên ta có $\frac{a}{a+b+c}<\frac{a}{b+c}<\frac{a+c}{a+b+c}$

 

$\Rightarrow \sum \frac{a}{a+b+c}<\sum \frac{a}{b+c}<\sum \frac{a+c}{a+b+c}$

 

Hay $1<\sum \frac{a}{b+c}<2$




#484152 $\boxed{\text{Chuyên Đề}}$ Bất đẳng thức - Cực trị

Đã gửi bởi angleofdarkness on 22-02-2014 - 18:06 trong Bất đẳng thức và cực trị

Đăng bài đi Việt Hoàng

 

Anh xử nốt bài 70 kia đi để em đăng bài lên vậy, làm cho xong rồi đăng bài mới luôn.




#484405 $\boxed{\text{Chuyên Đề}}$ Bất đẳng thức - Cực trị

Đã gửi bởi angleofdarkness on 23-02-2014 - 17:29 trong Bất đẳng thức và cực trị


80) Cho $a;b;c;p;q>0$. Cmr: $\sum \frac{a}{pb+qc}\geq \frac{3}{p+q}$

 

 

80/

 

C/m BĐT $(a+b+c)^2 \geq 3(ab+bc+ca)$ (*):

 

Từ BĐT $\sum a^2 \geq \sum ab$ (đã c/m) ta có $\sum a^2 + 2\sum ab \geq \sum ab + 2\sum ab$, tức là $(a+b+c)^2 \geq 3(ab+bc+ca)$

 

(*) đc c/m.

 

Ta có $\frac{a}{pb+qc}=\frac{a^2}{(pb+qc)a}=\frac{a^2}{pab+qca}$

 

$\Rightarrow  \sum \frac{a}{pb+qc}=\sum \frac{a^2}{pab+qca} \geq \frac{(a+b+c)^2}{pab+qca+pbc+qab+pca+qab}$ (BĐT Schwarz)

 

Hay $\sum \frac{a}{pb+qc} \geq \frac{(a+b+c)^2}{(ab+bc+ca)(p+q)}$ (nhóm các số ở mẫu vào theo từng cặp thích hợp)

 

Áp dụng BĐT (*) ta có $\sum \frac{a}{pb+qc} \geq \frac{3(ab+bc+ca)}{(ab+bc+ca)(p+q)}=\frac{3}{p+q}$

 

$\Rightarrow$ đpcm.

 

Dấu = khi a = b = c > 0.




#484384 $\boxed{\text{Chuyên Đề}}$ Bất đẳng thức - Cực trị

Đã gửi bởi angleofdarkness on 23-02-2014 - 16:28 trong Bất đẳng thức và cực trị

75) 

$x\leq 1\Rightarrow y\geq 2$

 

Dấu = có khi: $\left\{\begin{matrix}a=1 & & \\ b=-3 & & \end{matrix}\right.\Leftrightarrow \left\{\begin{matrix}x=0 & & \\ y=0 & & \end{matrix}\right.$

 

 

 

1. $a\geq 0; a+b\geq 0$ là do: $1-a\leq 1;a+b+2\geq 2$ . Nếu $b\geq 0$ mới là chưa chắc.

2. Dấu = đã đúng.

 

 

Không bảo sai,chỉ là không xảy ra dâu = thôi vì có $x \leq 1;y \geq 2$ nên không thể có x = y = 0.




#484203 $\boxed{\text{Chuyên Đề}}$ Bất đẳng thức - Cực trị

Đã gửi bởi angleofdarkness on 22-02-2014 - 21:07 trong Bất đẳng thức và cực trị

76) Cho $a+b\geq 2$. Cmr: $a^3+b^3\leq a^4+b^4$

 

 

Xét tổng $(a^4+b^4-a^3-b^3)+(2-a-b) \\ =a^4-a^3-a+1+b^4-b^3-b+1 \\ =(a-1)(a^3-1)+(b-1)(b^3-1) \\ =(a-1)^2(a^2+a+1)+(b-1)^2(b^2+b+1)$

 

Dễ thấy $a^2+a+1>0;b^2+b+1>0$ nên có $(a^4+b^4-a^3-b^3)+(2-a-b)\geq 0$

 

Mà có $a+b \geq 2$ nên $2-a-b \leq 0$ $\Rightarrow a^4+b^4-a^3-b^3 \geq 0 \Rightarrow a^4+b^4 \geq a^3-b^3$ (đpcm) 

 

Dấu = khi a = b = 1.

 

P/S: toàn quên cái dấu = nên phải sửa bài viết suốt  :lol: 




#484196 $\boxed{\text{Chuyên Đề}}$ Bất đẳng thức - Cực trị

Đã gửi bởi angleofdarkness on 22-02-2014 - 20:53 trong Bất đẳng thức và cực trị

Làm ngược từ dưới lên :D

 

 

 

78) Cho $a;b;c>0$ thỏa: $a+b+c=abc$. Cmr: $\sum \frac{a}{b^3}\geq 1$

 

 

Ta có $\frac{a}{b^3}+\frac{1}{ab} \geq \frac{2}{b^2}$ (Cauchy 2 số) $\Rightarrow \sum \frac{a}{b^3}+\sum \frac{1}{ab} \geq 2\sum \frac{1}{a^2}$ (1)

 

$\frac{1}{a^2}+\frac{1}{b^2} \geq \frac{2}{ab}\Rightarrow \sum \frac{1}{a^2} \geq \sum \frac{1}{ab}$ (2)

 

Mà có a + b + c = abc $\Rightarrow \sum \frac{1}{ab}=1$ 

 

Kết hợp (1) và (2) $\Rightarrow \sum \frac{a}{b^3} \geq 1$ (bước này hơi tắt)

 

Dấu = khi a = b = c = $\sqrt{3}$.




#481751 $\boxed{\text{Chuyên Đề}}$ Bất đẳng thức - Cực trị

Đã gửi bởi angleofdarkness on 07-02-2014 - 21:39 trong Bất đẳng thức và cực trị

8) Cmr: $\sum \frac{a^{3}}{b^{3}}\geq \sum \frac{a^{2}}{b^{2}}\forall a;b;c>0$

 

 

$a^3+b^3 \geq a^2b+ab^2 \Rightarrow \frac{a^3}{b^3}+1 \geq \frac{a^2}{b^2}+\frac{a}{b}$
 
$ \Rightarrow \sum \frac{a^3}{b^3} + 3 \geq \sum \frac{a^2}{b^2}+ \sum \frac{a}{b}$
 
Mà $ \sum \frac{a}{b} \geq 3$ nên ta có $ \sum \frac{a^3}{b^3} \geq \sum \frac{a^2}{b^2}$